Nonzero $f in C([0, 1])$ for which $int_0^1 f(x)x^n dx = 0$ for all $n$












23












$begingroup$


As the title says, I'm wondering if there is a continuous function such that $f$ is nonzero on $[0, 1]$, and for which $int_0^1 f(x)x^n dx = 0$ for all $n geq 1$. I am trying to solve a problem proving that if (on $C([0, 1])$) $int_0^1 f(x)x^n dx = 0$ for all $n geq 0$, then $f$ must be identically zero. I presume then we do require the $n=0$ case to hold too, otherwise it wouldn't be part of the statement. Is there ay function which is not identically zero which satisfies $int_0^1 f(x)x^n dx = 0$ for all $n geq 1$?



The statement I am attempting to prove is homework, but this is just idle curiosity (though I will tag it as homework anyway since it is related). Thank you!










share|cite|improve this question











$endgroup$








  • 1




    $begingroup$
    There are many counterexamples to both your claims. For example, the function $f$ with $f(0)=1$ and $f(x)=0$ for $x neq 0$. You'll need some additional assumption on $f$ to make them true.
    $endgroup$
    – Chris Eagle
    Jan 8 '11 at 22:04






  • 1




    $begingroup$
    @Chris: I think this is an exercise in Rudin or similar, and I think f is supposed to be continuous.
    $endgroup$
    – Qiaochu Yuan
    Jan 8 '11 at 22:08






  • 7




    $begingroup$
    Just apply your homework problem to $xf(x)$.
    $endgroup$
    – Zarrax
    Jan 8 '11 at 22:13






  • 1




    $begingroup$
    This maybe related.
    $endgroup$
    – timur
    Jan 9 '11 at 4:14






  • 2




    $begingroup$
    As an aside, the answer is yes if the interval is $(0,infty)$ instead of $(0,1)$. For example the "Stieltjes ghost function" $f(x) = exp(-x^{1/4}) sin x^{1/4}$ satisfies $int_0^{infty} f(x) x^n dx = 0$ for all integers $n ge 0$.
    $endgroup$
    – Hans Lundmark
    Jan 9 '11 at 10:18
















23












$begingroup$


As the title says, I'm wondering if there is a continuous function such that $f$ is nonzero on $[0, 1]$, and for which $int_0^1 f(x)x^n dx = 0$ for all $n geq 1$. I am trying to solve a problem proving that if (on $C([0, 1])$) $int_0^1 f(x)x^n dx = 0$ for all $n geq 0$, then $f$ must be identically zero. I presume then we do require the $n=0$ case to hold too, otherwise it wouldn't be part of the statement. Is there ay function which is not identically zero which satisfies $int_0^1 f(x)x^n dx = 0$ for all $n geq 1$?



The statement I am attempting to prove is homework, but this is just idle curiosity (though I will tag it as homework anyway since it is related). Thank you!










share|cite|improve this question











$endgroup$








  • 1




    $begingroup$
    There are many counterexamples to both your claims. For example, the function $f$ with $f(0)=1$ and $f(x)=0$ for $x neq 0$. You'll need some additional assumption on $f$ to make them true.
    $endgroup$
    – Chris Eagle
    Jan 8 '11 at 22:04






  • 1




    $begingroup$
    @Chris: I think this is an exercise in Rudin or similar, and I think f is supposed to be continuous.
    $endgroup$
    – Qiaochu Yuan
    Jan 8 '11 at 22:08






  • 7




    $begingroup$
    Just apply your homework problem to $xf(x)$.
    $endgroup$
    – Zarrax
    Jan 8 '11 at 22:13






  • 1




    $begingroup$
    This maybe related.
    $endgroup$
    – timur
    Jan 9 '11 at 4:14






  • 2




    $begingroup$
    As an aside, the answer is yes if the interval is $(0,infty)$ instead of $(0,1)$. For example the "Stieltjes ghost function" $f(x) = exp(-x^{1/4}) sin x^{1/4}$ satisfies $int_0^{infty} f(x) x^n dx = 0$ for all integers $n ge 0$.
    $endgroup$
    – Hans Lundmark
    Jan 9 '11 at 10:18














23












23








23


13



$begingroup$


As the title says, I'm wondering if there is a continuous function such that $f$ is nonzero on $[0, 1]$, and for which $int_0^1 f(x)x^n dx = 0$ for all $n geq 1$. I am trying to solve a problem proving that if (on $C([0, 1])$) $int_0^1 f(x)x^n dx = 0$ for all $n geq 0$, then $f$ must be identically zero. I presume then we do require the $n=0$ case to hold too, otherwise it wouldn't be part of the statement. Is there ay function which is not identically zero which satisfies $int_0^1 f(x)x^n dx = 0$ for all $n geq 1$?



The statement I am attempting to prove is homework, but this is just idle curiosity (though I will tag it as homework anyway since it is related). Thank you!










share|cite|improve this question











$endgroup$




As the title says, I'm wondering if there is a continuous function such that $f$ is nonzero on $[0, 1]$, and for which $int_0^1 f(x)x^n dx = 0$ for all $n geq 1$. I am trying to solve a problem proving that if (on $C([0, 1])$) $int_0^1 f(x)x^n dx = 0$ for all $n geq 0$, then $f$ must be identically zero. I presume then we do require the $n=0$ case to hold too, otherwise it wouldn't be part of the statement. Is there ay function which is not identically zero which satisfies $int_0^1 f(x)x^n dx = 0$ for all $n geq 1$?



The statement I am attempting to prove is homework, but this is just idle curiosity (though I will tag it as homework anyway since it is related). Thank you!







real-analysis analysis integration






share|cite|improve this question















share|cite|improve this question













share|cite|improve this question




share|cite|improve this question








edited Dec 20 '11 at 14:00









Srivatsan

21k371126




21k371126










asked Jan 8 '11 at 21:56









SpyamSpyam

1,63221426




1,63221426








  • 1




    $begingroup$
    There are many counterexamples to both your claims. For example, the function $f$ with $f(0)=1$ and $f(x)=0$ for $x neq 0$. You'll need some additional assumption on $f$ to make them true.
    $endgroup$
    – Chris Eagle
    Jan 8 '11 at 22:04






  • 1




    $begingroup$
    @Chris: I think this is an exercise in Rudin or similar, and I think f is supposed to be continuous.
    $endgroup$
    – Qiaochu Yuan
    Jan 8 '11 at 22:08






  • 7




    $begingroup$
    Just apply your homework problem to $xf(x)$.
    $endgroup$
    – Zarrax
    Jan 8 '11 at 22:13






  • 1




    $begingroup$
    This maybe related.
    $endgroup$
    – timur
    Jan 9 '11 at 4:14






  • 2




    $begingroup$
    As an aside, the answer is yes if the interval is $(0,infty)$ instead of $(0,1)$. For example the "Stieltjes ghost function" $f(x) = exp(-x^{1/4}) sin x^{1/4}$ satisfies $int_0^{infty} f(x) x^n dx = 0$ for all integers $n ge 0$.
    $endgroup$
    – Hans Lundmark
    Jan 9 '11 at 10:18














  • 1




    $begingroup$
    There are many counterexamples to both your claims. For example, the function $f$ with $f(0)=1$ and $f(x)=0$ for $x neq 0$. You'll need some additional assumption on $f$ to make them true.
    $endgroup$
    – Chris Eagle
    Jan 8 '11 at 22:04






  • 1




    $begingroup$
    @Chris: I think this is an exercise in Rudin or similar, and I think f is supposed to be continuous.
    $endgroup$
    – Qiaochu Yuan
    Jan 8 '11 at 22:08






  • 7




    $begingroup$
    Just apply your homework problem to $xf(x)$.
    $endgroup$
    – Zarrax
    Jan 8 '11 at 22:13






  • 1




    $begingroup$
    This maybe related.
    $endgroup$
    – timur
    Jan 9 '11 at 4:14






  • 2




    $begingroup$
    As an aside, the answer is yes if the interval is $(0,infty)$ instead of $(0,1)$. For example the "Stieltjes ghost function" $f(x) = exp(-x^{1/4}) sin x^{1/4}$ satisfies $int_0^{infty} f(x) x^n dx = 0$ for all integers $n ge 0$.
    $endgroup$
    – Hans Lundmark
    Jan 9 '11 at 10:18








1




1




$begingroup$
There are many counterexamples to both your claims. For example, the function $f$ with $f(0)=1$ and $f(x)=0$ for $x neq 0$. You'll need some additional assumption on $f$ to make them true.
$endgroup$
– Chris Eagle
Jan 8 '11 at 22:04




$begingroup$
There are many counterexamples to both your claims. For example, the function $f$ with $f(0)=1$ and $f(x)=0$ for $x neq 0$. You'll need some additional assumption on $f$ to make them true.
$endgroup$
– Chris Eagle
Jan 8 '11 at 22:04




1




1




$begingroup$
@Chris: I think this is an exercise in Rudin or similar, and I think f is supposed to be continuous.
$endgroup$
– Qiaochu Yuan
Jan 8 '11 at 22:08




$begingroup$
@Chris: I think this is an exercise in Rudin or similar, and I think f is supposed to be continuous.
$endgroup$
– Qiaochu Yuan
Jan 8 '11 at 22:08




7




7




$begingroup$
Just apply your homework problem to $xf(x)$.
$endgroup$
– Zarrax
Jan 8 '11 at 22:13




$begingroup$
Just apply your homework problem to $xf(x)$.
$endgroup$
– Zarrax
Jan 8 '11 at 22:13




1




1




$begingroup$
This maybe related.
$endgroup$
– timur
Jan 9 '11 at 4:14




$begingroup$
This maybe related.
$endgroup$
– timur
Jan 9 '11 at 4:14




2




2




$begingroup$
As an aside, the answer is yes if the interval is $(0,infty)$ instead of $(0,1)$. For example the "Stieltjes ghost function" $f(x) = exp(-x^{1/4}) sin x^{1/4}$ satisfies $int_0^{infty} f(x) x^n dx = 0$ for all integers $n ge 0$.
$endgroup$
– Hans Lundmark
Jan 9 '11 at 10:18




$begingroup$
As an aside, the answer is yes if the interval is $(0,infty)$ instead of $(0,1)$. For example the "Stieltjes ghost function" $f(x) = exp(-x^{1/4}) sin x^{1/4}$ satisfies $int_0^{infty} f(x) x^n dx = 0$ for all integers $n ge 0$.
$endgroup$
– Hans Lundmark
Jan 9 '11 at 10:18










5 Answers
5






active

oldest

votes


















6












$begingroup$

(I am turning this into Community wiki, since the original version made an obvious mistake).



The result follows, for example, from the Stone-Weierstrass theorem, once one justifies that the limit of some integrals is the integral of the limit, which can be done (overkill) using Lebesgue's dominated convergence theorem or (more easily) using simple estimates from the fact that $f$ is bounded, since it is continuous.



Below I give full details, which you should probably not read until after your homework is due, since this also solves your homework.





Spoilers:



There is a sequence of polynomials $p_n(x)$ that converges uniformly to $xf(x)$ on ${}[0,1]$. We have $int_0^1xf(x)p_n(x)dx=0$ for all $n$, by assumption, since $xp_n(x)$ is a sum of monomials the integral of whose integral with $f$ is 0. Now take the limit as $ntoinfty$ to conclude that $int_0^1x(f(x))^2dx=0$.



This gives us that $f=0$ because if $f(x_0)ne 0$, continuity ensures a positive $epsilon>0$ and an interval $(a,b)$ with $a>0$ such that $|f(x)|geepsilon$ for all $xin(a,b)$. But then $int_0^1xf(x)^2dxge laepsilon^2>0$, where $l=b-a$ is the length of the interval.



To see that the limit of the integrals is 0 without using dominated convergence, let $Mge|f(x)|$ for all $xin[0,1]$. The, for any $delta>0$, if $n$ is large enough, we have $$int_0^1f(x)xp_n(x)dx=int_0^1ftimes(p-xf+xf)dx=int_0^1xf(x)^2dx+int_0^1ftimes(p-xf)dx,$$ and the second integral is bounded by $int_0^1|f||p-xf|dxle M(delta/M)=delta$.



In fact, even this is approach is an overkill. (For example, Müntz's theorem gives a more general fact, as already mentioned in another answer.)



(Apologies for the original mistake.)






share|cite|improve this answer











$endgroup$









  • 1




    $begingroup$
    The Stone-Weierstrass theorem doesn't (directly) apply to the question the OP asked, since the subalgebra generated by {x^n | n > 0} doesn't contain a constant function. But Zarrax's comment takes care of this.
    $endgroup$
    – Qiaochu Yuan
    Jan 8 '11 at 22:28










  • $begingroup$
    I would say Muntz theorem is overkill! (and not your current answer).
    $endgroup$
    – Aryabhata
    Jan 9 '11 at 0:57












  • $begingroup$
    :-) I meant overkill in the sense that we are using hypothesis that are way too strong for the task at hand. (It would be nice to try to weaken them significantly.)
    $endgroup$
    – Andrés E. Caicedo
    Jan 9 '11 at 2:39



















15












$begingroup$

As an aside, the answer is yes if the interval is $(0,infty)$ instead of $(0,1)$.
For example the "Stieltjes ghost function"
$f(x) = exp(-x^{1/4}) sin x^{1/4}$ satisfies
$int_0^{infty} f(x) x^n dx = 0$
for all integers $n ge 0$.



Stieltjes gave this as an example of a case where the
moment problem
does not have a unique solution.
It appears in Section 55 of his famous paper "Recherches sur les fractions continues" from 1894;
see p. 506 in
Œuvres Complètes, Vol. II.
To compute the moments, use the substitution $x=u^4$ to write
$I_n = int_0^{infty} f(x) x^n dx = 4 int_0^{infty} e^{-u} sin(u) u^{4n+3} du$;
then integrate by parts four times (differentiating the power of $u$, and integrating the rest) to show that $I_n$ is proportional to $I_{n-1}$,
and finally check that $I_0=0$.






share|cite|improve this answer









$endgroup$





















    7












    $begingroup$

    The answer is no. Actually I believe the following is a theorem whose name totally escapes me at the moment: assume that $f$ is continuous and let $a_n$ be a sequence of increasing positive integers such that $int_0^1 f(x) x^{a_n} , dx = 0$ for $n ge 1$. If $sum frac{1}{a_n}$ diverges, then $f$ is identically zero! (Edit: this is a corollary of the Müntz–Szász theorem - thanks, Moron!)



    In other words, the problem isn't phrased the way it is because stronger statements are false; the stronger versions are just harder to prove.






    share|cite|improve this answer











    $endgroup$













    • $begingroup$
      Corollary of Muntz theorem, I believe.
      $endgroup$
      – Aryabhata
      Jan 8 '11 at 22:12












    • $begingroup$
      +1, but that is actually not needed :-) (see Zarrax's comment).
      $endgroup$
      – Aryabhata
      Jan 8 '11 at 22:22






    • 6




      $begingroup$
      +1, interesting theorem. As a side note, this is one of those posts where now that Moron has changed his name, the way it reads is rather funny.
      $endgroup$
      – Eric Naslund
      Dec 20 '11 at 12:57



















    5












    $begingroup$

    There is a proof, using the Weierstrass approximation theorem, that if $f$ is continuous then $f$ is necessarily zero!



    Classical Weierstrass's Theorem : If f is a continuous real valued function on $[a, b]$, then there exists a sequence of polynomials $p_n$ such that
    $$ lim_{nrightarrow+infty}p_n(x)=f(x)$$ uniformly on $[a, b]$
    .



    By the assumptions of the problem and the linearity of the integral
    is easy to see that
    $$
    int_{0}^1f(x)p(x)dx=0
    $$
    for all polynomials $p(x)$ on $C([0,1])$. Now just apply Theorem!






    share|cite|improve this answer











    $endgroup$





















      1












      $begingroup$

      Just for fun, here's a proof in non-standard analysis (Nelson-style IST). I write $a approx b$ to mean that $a-b$ is infinitesimal.



      It's enough to prove the result when $f$ is standard. The Weierstrass approximation theorem gives a polynomial $p$ such that $p(x) approx x f(x)$ for all $x in [0,1]$. Note $int _0 ^1 xfp = 0$, because the integral is linear.



      Now estimate the absolute value of $int xfp - (xf)^2$. Factoring the integrand as $xf(x)cdot ( p(x) - xf(x) )$, we see that this integral is bounded above by $operatorname{sup}(xf) cdot operatorname{sup}(p-xf)$, where the sup is over all $x in [0,1]$. This is infinitesimal, since the sup of a standard continuous function on a standard compact interval is limited.



      We therefore have $int (xf)^2 approx int xfp = 0$. Since $int (xf)^2$ is standard, it is equal to zero. Therefore $(xf)^2$ is identically zero, and so is $f$ (being continuous).






      share|cite|improve this answer











      $endgroup$













        Your Answer





        StackExchange.ifUsing("editor", function () {
        return StackExchange.using("mathjaxEditing", function () {
        StackExchange.MarkdownEditor.creationCallbacks.add(function (editor, postfix) {
        StackExchange.mathjaxEditing.prepareWmdForMathJax(editor, postfix, [["$", "$"], ["\\(","\\)"]]);
        });
        });
        }, "mathjax-editing");

        StackExchange.ready(function() {
        var channelOptions = {
        tags: "".split(" "),
        id: "69"
        };
        initTagRenderer("".split(" "), "".split(" "), channelOptions);

        StackExchange.using("externalEditor", function() {
        // Have to fire editor after snippets, if snippets enabled
        if (StackExchange.settings.snippets.snippetsEnabled) {
        StackExchange.using("snippets", function() {
        createEditor();
        });
        }
        else {
        createEditor();
        }
        });

        function createEditor() {
        StackExchange.prepareEditor({
        heartbeatType: 'answer',
        autoActivateHeartbeat: false,
        convertImagesToLinks: true,
        noModals: true,
        showLowRepImageUploadWarning: true,
        reputationToPostImages: 10,
        bindNavPrevention: true,
        postfix: "",
        imageUploader: {
        brandingHtml: "Powered by u003ca class="icon-imgur-white" href="https://imgur.com/"u003eu003c/au003e",
        contentPolicyHtml: "User contributions licensed under u003ca href="https://creativecommons.org/licenses/by-sa/3.0/"u003ecc by-sa 3.0 with attribution requiredu003c/au003e u003ca href="https://stackoverflow.com/legal/content-policy"u003e(content policy)u003c/au003e",
        allowUrls: true
        },
        noCode: true, onDemand: true,
        discardSelector: ".discard-answer"
        ,immediatelyShowMarkdownHelp:true
        });


        }
        });














        draft saved

        draft discarded


















        StackExchange.ready(
        function () {
        StackExchange.openid.initPostLogin('.new-post-login', 'https%3a%2f%2fmath.stackexchange.com%2fquestions%2f16831%2fnonzero-f-in-c0-1-for-which-int-01-fxxn-dx-0-for-all-n%23new-answer', 'question_page');
        }
        );

        Post as a guest















        Required, but never shown

























        5 Answers
        5






        active

        oldest

        votes








        5 Answers
        5






        active

        oldest

        votes









        active

        oldest

        votes






        active

        oldest

        votes









        6












        $begingroup$

        (I am turning this into Community wiki, since the original version made an obvious mistake).



        The result follows, for example, from the Stone-Weierstrass theorem, once one justifies that the limit of some integrals is the integral of the limit, which can be done (overkill) using Lebesgue's dominated convergence theorem or (more easily) using simple estimates from the fact that $f$ is bounded, since it is continuous.



        Below I give full details, which you should probably not read until after your homework is due, since this also solves your homework.





        Spoilers:



        There is a sequence of polynomials $p_n(x)$ that converges uniformly to $xf(x)$ on ${}[0,1]$. We have $int_0^1xf(x)p_n(x)dx=0$ for all $n$, by assumption, since $xp_n(x)$ is a sum of monomials the integral of whose integral with $f$ is 0. Now take the limit as $ntoinfty$ to conclude that $int_0^1x(f(x))^2dx=0$.



        This gives us that $f=0$ because if $f(x_0)ne 0$, continuity ensures a positive $epsilon>0$ and an interval $(a,b)$ with $a>0$ such that $|f(x)|geepsilon$ for all $xin(a,b)$. But then $int_0^1xf(x)^2dxge laepsilon^2>0$, where $l=b-a$ is the length of the interval.



        To see that the limit of the integrals is 0 without using dominated convergence, let $Mge|f(x)|$ for all $xin[0,1]$. The, for any $delta>0$, if $n$ is large enough, we have $$int_0^1f(x)xp_n(x)dx=int_0^1ftimes(p-xf+xf)dx=int_0^1xf(x)^2dx+int_0^1ftimes(p-xf)dx,$$ and the second integral is bounded by $int_0^1|f||p-xf|dxle M(delta/M)=delta$.



        In fact, even this is approach is an overkill. (For example, Müntz's theorem gives a more general fact, as already mentioned in another answer.)



        (Apologies for the original mistake.)






        share|cite|improve this answer











        $endgroup$









        • 1




          $begingroup$
          The Stone-Weierstrass theorem doesn't (directly) apply to the question the OP asked, since the subalgebra generated by {x^n | n > 0} doesn't contain a constant function. But Zarrax's comment takes care of this.
          $endgroup$
          – Qiaochu Yuan
          Jan 8 '11 at 22:28










        • $begingroup$
          I would say Muntz theorem is overkill! (and not your current answer).
          $endgroup$
          – Aryabhata
          Jan 9 '11 at 0:57












        • $begingroup$
          :-) I meant overkill in the sense that we are using hypothesis that are way too strong for the task at hand. (It would be nice to try to weaken them significantly.)
          $endgroup$
          – Andrés E. Caicedo
          Jan 9 '11 at 2:39
















        6












        $begingroup$

        (I am turning this into Community wiki, since the original version made an obvious mistake).



        The result follows, for example, from the Stone-Weierstrass theorem, once one justifies that the limit of some integrals is the integral of the limit, which can be done (overkill) using Lebesgue's dominated convergence theorem or (more easily) using simple estimates from the fact that $f$ is bounded, since it is continuous.



        Below I give full details, which you should probably not read until after your homework is due, since this also solves your homework.





        Spoilers:



        There is a sequence of polynomials $p_n(x)$ that converges uniformly to $xf(x)$ on ${}[0,1]$. We have $int_0^1xf(x)p_n(x)dx=0$ for all $n$, by assumption, since $xp_n(x)$ is a sum of monomials the integral of whose integral with $f$ is 0. Now take the limit as $ntoinfty$ to conclude that $int_0^1x(f(x))^2dx=0$.



        This gives us that $f=0$ because if $f(x_0)ne 0$, continuity ensures a positive $epsilon>0$ and an interval $(a,b)$ with $a>0$ such that $|f(x)|geepsilon$ for all $xin(a,b)$. But then $int_0^1xf(x)^2dxge laepsilon^2>0$, where $l=b-a$ is the length of the interval.



        To see that the limit of the integrals is 0 without using dominated convergence, let $Mge|f(x)|$ for all $xin[0,1]$. The, for any $delta>0$, if $n$ is large enough, we have $$int_0^1f(x)xp_n(x)dx=int_0^1ftimes(p-xf+xf)dx=int_0^1xf(x)^2dx+int_0^1ftimes(p-xf)dx,$$ and the second integral is bounded by $int_0^1|f||p-xf|dxle M(delta/M)=delta$.



        In fact, even this is approach is an overkill. (For example, Müntz's theorem gives a more general fact, as already mentioned in another answer.)



        (Apologies for the original mistake.)






        share|cite|improve this answer











        $endgroup$









        • 1




          $begingroup$
          The Stone-Weierstrass theorem doesn't (directly) apply to the question the OP asked, since the subalgebra generated by {x^n | n > 0} doesn't contain a constant function. But Zarrax's comment takes care of this.
          $endgroup$
          – Qiaochu Yuan
          Jan 8 '11 at 22:28










        • $begingroup$
          I would say Muntz theorem is overkill! (and not your current answer).
          $endgroup$
          – Aryabhata
          Jan 9 '11 at 0:57












        • $begingroup$
          :-) I meant overkill in the sense that we are using hypothesis that are way too strong for the task at hand. (It would be nice to try to weaken them significantly.)
          $endgroup$
          – Andrés E. Caicedo
          Jan 9 '11 at 2:39














        6












        6








        6





        $begingroup$

        (I am turning this into Community wiki, since the original version made an obvious mistake).



        The result follows, for example, from the Stone-Weierstrass theorem, once one justifies that the limit of some integrals is the integral of the limit, which can be done (overkill) using Lebesgue's dominated convergence theorem or (more easily) using simple estimates from the fact that $f$ is bounded, since it is continuous.



        Below I give full details, which you should probably not read until after your homework is due, since this also solves your homework.





        Spoilers:



        There is a sequence of polynomials $p_n(x)$ that converges uniformly to $xf(x)$ on ${}[0,1]$. We have $int_0^1xf(x)p_n(x)dx=0$ for all $n$, by assumption, since $xp_n(x)$ is a sum of monomials the integral of whose integral with $f$ is 0. Now take the limit as $ntoinfty$ to conclude that $int_0^1x(f(x))^2dx=0$.



        This gives us that $f=0$ because if $f(x_0)ne 0$, continuity ensures a positive $epsilon>0$ and an interval $(a,b)$ with $a>0$ such that $|f(x)|geepsilon$ for all $xin(a,b)$. But then $int_0^1xf(x)^2dxge laepsilon^2>0$, where $l=b-a$ is the length of the interval.



        To see that the limit of the integrals is 0 without using dominated convergence, let $Mge|f(x)|$ for all $xin[0,1]$. The, for any $delta>0$, if $n$ is large enough, we have $$int_0^1f(x)xp_n(x)dx=int_0^1ftimes(p-xf+xf)dx=int_0^1xf(x)^2dx+int_0^1ftimes(p-xf)dx,$$ and the second integral is bounded by $int_0^1|f||p-xf|dxle M(delta/M)=delta$.



        In fact, even this is approach is an overkill. (For example, Müntz's theorem gives a more general fact, as already mentioned in another answer.)



        (Apologies for the original mistake.)






        share|cite|improve this answer











        $endgroup$



        (I am turning this into Community wiki, since the original version made an obvious mistake).



        The result follows, for example, from the Stone-Weierstrass theorem, once one justifies that the limit of some integrals is the integral of the limit, which can be done (overkill) using Lebesgue's dominated convergence theorem or (more easily) using simple estimates from the fact that $f$ is bounded, since it is continuous.



        Below I give full details, which you should probably not read until after your homework is due, since this also solves your homework.





        Spoilers:



        There is a sequence of polynomials $p_n(x)$ that converges uniformly to $xf(x)$ on ${}[0,1]$. We have $int_0^1xf(x)p_n(x)dx=0$ for all $n$, by assumption, since $xp_n(x)$ is a sum of monomials the integral of whose integral with $f$ is 0. Now take the limit as $ntoinfty$ to conclude that $int_0^1x(f(x))^2dx=0$.



        This gives us that $f=0$ because if $f(x_0)ne 0$, continuity ensures a positive $epsilon>0$ and an interval $(a,b)$ with $a>0$ such that $|f(x)|geepsilon$ for all $xin(a,b)$. But then $int_0^1xf(x)^2dxge laepsilon^2>0$, where $l=b-a$ is the length of the interval.



        To see that the limit of the integrals is 0 without using dominated convergence, let $Mge|f(x)|$ for all $xin[0,1]$. The, for any $delta>0$, if $n$ is large enough, we have $$int_0^1f(x)xp_n(x)dx=int_0^1ftimes(p-xf+xf)dx=int_0^1xf(x)^2dx+int_0^1ftimes(p-xf)dx,$$ and the second integral is bounded by $int_0^1|f||p-xf|dxle M(delta/M)=delta$.



        In fact, even this is approach is an overkill. (For example, Müntz's theorem gives a more general fact, as already mentioned in another answer.)



        (Apologies for the original mistake.)







        share|cite|improve this answer














        share|cite|improve this answer



        share|cite|improve this answer








        edited Jan 8 '11 at 23:58


























        community wiki





        4 revs
        Andres Caicedo









        • 1




          $begingroup$
          The Stone-Weierstrass theorem doesn't (directly) apply to the question the OP asked, since the subalgebra generated by {x^n | n > 0} doesn't contain a constant function. But Zarrax's comment takes care of this.
          $endgroup$
          – Qiaochu Yuan
          Jan 8 '11 at 22:28










        • $begingroup$
          I would say Muntz theorem is overkill! (and not your current answer).
          $endgroup$
          – Aryabhata
          Jan 9 '11 at 0:57












        • $begingroup$
          :-) I meant overkill in the sense that we are using hypothesis that are way too strong for the task at hand. (It would be nice to try to weaken them significantly.)
          $endgroup$
          – Andrés E. Caicedo
          Jan 9 '11 at 2:39














        • 1




          $begingroup$
          The Stone-Weierstrass theorem doesn't (directly) apply to the question the OP asked, since the subalgebra generated by {x^n | n > 0} doesn't contain a constant function. But Zarrax's comment takes care of this.
          $endgroup$
          – Qiaochu Yuan
          Jan 8 '11 at 22:28










        • $begingroup$
          I would say Muntz theorem is overkill! (and not your current answer).
          $endgroup$
          – Aryabhata
          Jan 9 '11 at 0:57












        • $begingroup$
          :-) I meant overkill in the sense that we are using hypothesis that are way too strong for the task at hand. (It would be nice to try to weaken them significantly.)
          $endgroup$
          – Andrés E. Caicedo
          Jan 9 '11 at 2:39








        1




        1




        $begingroup$
        The Stone-Weierstrass theorem doesn't (directly) apply to the question the OP asked, since the subalgebra generated by {x^n | n > 0} doesn't contain a constant function. But Zarrax's comment takes care of this.
        $endgroup$
        – Qiaochu Yuan
        Jan 8 '11 at 22:28




        $begingroup$
        The Stone-Weierstrass theorem doesn't (directly) apply to the question the OP asked, since the subalgebra generated by {x^n | n > 0} doesn't contain a constant function. But Zarrax's comment takes care of this.
        $endgroup$
        – Qiaochu Yuan
        Jan 8 '11 at 22:28












        $begingroup$
        I would say Muntz theorem is overkill! (and not your current answer).
        $endgroup$
        – Aryabhata
        Jan 9 '11 at 0:57






        $begingroup$
        I would say Muntz theorem is overkill! (and not your current answer).
        $endgroup$
        – Aryabhata
        Jan 9 '11 at 0:57














        $begingroup$
        :-) I meant overkill in the sense that we are using hypothesis that are way too strong for the task at hand. (It would be nice to try to weaken them significantly.)
        $endgroup$
        – Andrés E. Caicedo
        Jan 9 '11 at 2:39




        $begingroup$
        :-) I meant overkill in the sense that we are using hypothesis that are way too strong for the task at hand. (It would be nice to try to weaken them significantly.)
        $endgroup$
        – Andrés E. Caicedo
        Jan 9 '11 at 2:39











        15












        $begingroup$

        As an aside, the answer is yes if the interval is $(0,infty)$ instead of $(0,1)$.
        For example the "Stieltjes ghost function"
        $f(x) = exp(-x^{1/4}) sin x^{1/4}$ satisfies
        $int_0^{infty} f(x) x^n dx = 0$
        for all integers $n ge 0$.



        Stieltjes gave this as an example of a case where the
        moment problem
        does not have a unique solution.
        It appears in Section 55 of his famous paper "Recherches sur les fractions continues" from 1894;
        see p. 506 in
        Œuvres Complètes, Vol. II.
        To compute the moments, use the substitution $x=u^4$ to write
        $I_n = int_0^{infty} f(x) x^n dx = 4 int_0^{infty} e^{-u} sin(u) u^{4n+3} du$;
        then integrate by parts four times (differentiating the power of $u$, and integrating the rest) to show that $I_n$ is proportional to $I_{n-1}$,
        and finally check that $I_0=0$.






        share|cite|improve this answer









        $endgroup$


















          15












          $begingroup$

          As an aside, the answer is yes if the interval is $(0,infty)$ instead of $(0,1)$.
          For example the "Stieltjes ghost function"
          $f(x) = exp(-x^{1/4}) sin x^{1/4}$ satisfies
          $int_0^{infty} f(x) x^n dx = 0$
          for all integers $n ge 0$.



          Stieltjes gave this as an example of a case where the
          moment problem
          does not have a unique solution.
          It appears in Section 55 of his famous paper "Recherches sur les fractions continues" from 1894;
          see p. 506 in
          Œuvres Complètes, Vol. II.
          To compute the moments, use the substitution $x=u^4$ to write
          $I_n = int_0^{infty} f(x) x^n dx = 4 int_0^{infty} e^{-u} sin(u) u^{4n+3} du$;
          then integrate by parts four times (differentiating the power of $u$, and integrating the rest) to show that $I_n$ is proportional to $I_{n-1}$,
          and finally check that $I_0=0$.






          share|cite|improve this answer









          $endgroup$
















            15












            15








            15





            $begingroup$

            As an aside, the answer is yes if the interval is $(0,infty)$ instead of $(0,1)$.
            For example the "Stieltjes ghost function"
            $f(x) = exp(-x^{1/4}) sin x^{1/4}$ satisfies
            $int_0^{infty} f(x) x^n dx = 0$
            for all integers $n ge 0$.



            Stieltjes gave this as an example of a case where the
            moment problem
            does not have a unique solution.
            It appears in Section 55 of his famous paper "Recherches sur les fractions continues" from 1894;
            see p. 506 in
            Œuvres Complètes, Vol. II.
            To compute the moments, use the substitution $x=u^4$ to write
            $I_n = int_0^{infty} f(x) x^n dx = 4 int_0^{infty} e^{-u} sin(u) u^{4n+3} du$;
            then integrate by parts four times (differentiating the power of $u$, and integrating the rest) to show that $I_n$ is proportional to $I_{n-1}$,
            and finally check that $I_0=0$.






            share|cite|improve this answer









            $endgroup$



            As an aside, the answer is yes if the interval is $(0,infty)$ instead of $(0,1)$.
            For example the "Stieltjes ghost function"
            $f(x) = exp(-x^{1/4}) sin x^{1/4}$ satisfies
            $int_0^{infty} f(x) x^n dx = 0$
            for all integers $n ge 0$.



            Stieltjes gave this as an example of a case where the
            moment problem
            does not have a unique solution.
            It appears in Section 55 of his famous paper "Recherches sur les fractions continues" from 1894;
            see p. 506 in
            Œuvres Complètes, Vol. II.
            To compute the moments, use the substitution $x=u^4$ to write
            $I_n = int_0^{infty} f(x) x^n dx = 4 int_0^{infty} e^{-u} sin(u) u^{4n+3} du$;
            then integrate by parts four times (differentiating the power of $u$, and integrating the rest) to show that $I_n$ is proportional to $I_{n-1}$,
            and finally check that $I_0=0$.







            share|cite|improve this answer












            share|cite|improve this answer



            share|cite|improve this answer










            answered Jan 10 '11 at 23:23









            Hans LundmarkHans Lundmark

            35.9k564115




            35.9k564115























                7












                $begingroup$

                The answer is no. Actually I believe the following is a theorem whose name totally escapes me at the moment: assume that $f$ is continuous and let $a_n$ be a sequence of increasing positive integers such that $int_0^1 f(x) x^{a_n} , dx = 0$ for $n ge 1$. If $sum frac{1}{a_n}$ diverges, then $f$ is identically zero! (Edit: this is a corollary of the Müntz–Szász theorem - thanks, Moron!)



                In other words, the problem isn't phrased the way it is because stronger statements are false; the stronger versions are just harder to prove.






                share|cite|improve this answer











                $endgroup$













                • $begingroup$
                  Corollary of Muntz theorem, I believe.
                  $endgroup$
                  – Aryabhata
                  Jan 8 '11 at 22:12












                • $begingroup$
                  +1, but that is actually not needed :-) (see Zarrax's comment).
                  $endgroup$
                  – Aryabhata
                  Jan 8 '11 at 22:22






                • 6




                  $begingroup$
                  +1, interesting theorem. As a side note, this is one of those posts where now that Moron has changed his name, the way it reads is rather funny.
                  $endgroup$
                  – Eric Naslund
                  Dec 20 '11 at 12:57
















                7












                $begingroup$

                The answer is no. Actually I believe the following is a theorem whose name totally escapes me at the moment: assume that $f$ is continuous and let $a_n$ be a sequence of increasing positive integers such that $int_0^1 f(x) x^{a_n} , dx = 0$ for $n ge 1$. If $sum frac{1}{a_n}$ diverges, then $f$ is identically zero! (Edit: this is a corollary of the Müntz–Szász theorem - thanks, Moron!)



                In other words, the problem isn't phrased the way it is because stronger statements are false; the stronger versions are just harder to prove.






                share|cite|improve this answer











                $endgroup$













                • $begingroup$
                  Corollary of Muntz theorem, I believe.
                  $endgroup$
                  – Aryabhata
                  Jan 8 '11 at 22:12












                • $begingroup$
                  +1, but that is actually not needed :-) (see Zarrax's comment).
                  $endgroup$
                  – Aryabhata
                  Jan 8 '11 at 22:22






                • 6




                  $begingroup$
                  +1, interesting theorem. As a side note, this is one of those posts where now that Moron has changed his name, the way it reads is rather funny.
                  $endgroup$
                  – Eric Naslund
                  Dec 20 '11 at 12:57














                7












                7








                7





                $begingroup$

                The answer is no. Actually I believe the following is a theorem whose name totally escapes me at the moment: assume that $f$ is continuous and let $a_n$ be a sequence of increasing positive integers such that $int_0^1 f(x) x^{a_n} , dx = 0$ for $n ge 1$. If $sum frac{1}{a_n}$ diverges, then $f$ is identically zero! (Edit: this is a corollary of the Müntz–Szász theorem - thanks, Moron!)



                In other words, the problem isn't phrased the way it is because stronger statements are false; the stronger versions are just harder to prove.






                share|cite|improve this answer











                $endgroup$



                The answer is no. Actually I believe the following is a theorem whose name totally escapes me at the moment: assume that $f$ is continuous and let $a_n$ be a sequence of increasing positive integers such that $int_0^1 f(x) x^{a_n} , dx = 0$ for $n ge 1$. If $sum frac{1}{a_n}$ diverges, then $f$ is identically zero! (Edit: this is a corollary of the Müntz–Szász theorem - thanks, Moron!)



                In other words, the problem isn't phrased the way it is because stronger statements are false; the stronger versions are just harder to prove.







                share|cite|improve this answer














                share|cite|improve this answer



                share|cite|improve this answer








                edited Jan 8 '11 at 22:16

























                answered Jan 8 '11 at 22:11









                Qiaochu YuanQiaochu Yuan

                281k32593939




                281k32593939












                • $begingroup$
                  Corollary of Muntz theorem, I believe.
                  $endgroup$
                  – Aryabhata
                  Jan 8 '11 at 22:12












                • $begingroup$
                  +1, but that is actually not needed :-) (see Zarrax's comment).
                  $endgroup$
                  – Aryabhata
                  Jan 8 '11 at 22:22






                • 6




                  $begingroup$
                  +1, interesting theorem. As a side note, this is one of those posts where now that Moron has changed his name, the way it reads is rather funny.
                  $endgroup$
                  – Eric Naslund
                  Dec 20 '11 at 12:57


















                • $begingroup$
                  Corollary of Muntz theorem, I believe.
                  $endgroup$
                  – Aryabhata
                  Jan 8 '11 at 22:12












                • $begingroup$
                  +1, but that is actually not needed :-) (see Zarrax's comment).
                  $endgroup$
                  – Aryabhata
                  Jan 8 '11 at 22:22






                • 6




                  $begingroup$
                  +1, interesting theorem. As a side note, this is one of those posts where now that Moron has changed his name, the way it reads is rather funny.
                  $endgroup$
                  – Eric Naslund
                  Dec 20 '11 at 12:57
















                $begingroup$
                Corollary of Muntz theorem, I believe.
                $endgroup$
                – Aryabhata
                Jan 8 '11 at 22:12






                $begingroup$
                Corollary of Muntz theorem, I believe.
                $endgroup$
                – Aryabhata
                Jan 8 '11 at 22:12














                $begingroup$
                +1, but that is actually not needed :-) (see Zarrax's comment).
                $endgroup$
                – Aryabhata
                Jan 8 '11 at 22:22




                $begingroup$
                +1, but that is actually not needed :-) (see Zarrax's comment).
                $endgroup$
                – Aryabhata
                Jan 8 '11 at 22:22




                6




                6




                $begingroup$
                +1, interesting theorem. As a side note, this is one of those posts where now that Moron has changed his name, the way it reads is rather funny.
                $endgroup$
                – Eric Naslund
                Dec 20 '11 at 12:57




                $begingroup$
                +1, interesting theorem. As a side note, this is one of those posts where now that Moron has changed his name, the way it reads is rather funny.
                $endgroup$
                – Eric Naslund
                Dec 20 '11 at 12:57











                5












                $begingroup$

                There is a proof, using the Weierstrass approximation theorem, that if $f$ is continuous then $f$ is necessarily zero!



                Classical Weierstrass's Theorem : If f is a continuous real valued function on $[a, b]$, then there exists a sequence of polynomials $p_n$ such that
                $$ lim_{nrightarrow+infty}p_n(x)=f(x)$$ uniformly on $[a, b]$
                .



                By the assumptions of the problem and the linearity of the integral
                is easy to see that
                $$
                int_{0}^1f(x)p(x)dx=0
                $$
                for all polynomials $p(x)$ on $C([0,1])$. Now just apply Theorem!






                share|cite|improve this answer











                $endgroup$


















                  5












                  $begingroup$

                  There is a proof, using the Weierstrass approximation theorem, that if $f$ is continuous then $f$ is necessarily zero!



                  Classical Weierstrass's Theorem : If f is a continuous real valued function on $[a, b]$, then there exists a sequence of polynomials $p_n$ such that
                  $$ lim_{nrightarrow+infty}p_n(x)=f(x)$$ uniformly on $[a, b]$
                  .



                  By the assumptions of the problem and the linearity of the integral
                  is easy to see that
                  $$
                  int_{0}^1f(x)p(x)dx=0
                  $$
                  for all polynomials $p(x)$ on $C([0,1])$. Now just apply Theorem!






                  share|cite|improve this answer











                  $endgroup$
















                    5












                    5








                    5





                    $begingroup$

                    There is a proof, using the Weierstrass approximation theorem, that if $f$ is continuous then $f$ is necessarily zero!



                    Classical Weierstrass's Theorem : If f is a continuous real valued function on $[a, b]$, then there exists a sequence of polynomials $p_n$ such that
                    $$ lim_{nrightarrow+infty}p_n(x)=f(x)$$ uniformly on $[a, b]$
                    .



                    By the assumptions of the problem and the linearity of the integral
                    is easy to see that
                    $$
                    int_{0}^1f(x)p(x)dx=0
                    $$
                    for all polynomials $p(x)$ on $C([0,1])$. Now just apply Theorem!






                    share|cite|improve this answer











                    $endgroup$



                    There is a proof, using the Weierstrass approximation theorem, that if $f$ is continuous then $f$ is necessarily zero!



                    Classical Weierstrass's Theorem : If f is a continuous real valued function on $[a, b]$, then there exists a sequence of polynomials $p_n$ such that
                    $$ lim_{nrightarrow+infty}p_n(x)=f(x)$$ uniformly on $[a, b]$
                    .



                    By the assumptions of the problem and the linearity of the integral
                    is easy to see that
                    $$
                    int_{0}^1f(x)p(x)dx=0
                    $$
                    for all polynomials $p(x)$ on $C([0,1])$. Now just apply Theorem!







                    share|cite|improve this answer














                    share|cite|improve this answer



                    share|cite|improve this answer








                    edited Dec 20 '11 at 14:02









                    Srivatsan

                    21k371126




                    21k371126










                    answered Dec 20 '11 at 12:38









                    MathOverviewMathOverview

                    8,96043164




                    8,96043164























                        1












                        $begingroup$

                        Just for fun, here's a proof in non-standard analysis (Nelson-style IST). I write $a approx b$ to mean that $a-b$ is infinitesimal.



                        It's enough to prove the result when $f$ is standard. The Weierstrass approximation theorem gives a polynomial $p$ such that $p(x) approx x f(x)$ for all $x in [0,1]$. Note $int _0 ^1 xfp = 0$, because the integral is linear.



                        Now estimate the absolute value of $int xfp - (xf)^2$. Factoring the integrand as $xf(x)cdot ( p(x) - xf(x) )$, we see that this integral is bounded above by $operatorname{sup}(xf) cdot operatorname{sup}(p-xf)$, where the sup is over all $x in [0,1]$. This is infinitesimal, since the sup of a standard continuous function on a standard compact interval is limited.



                        We therefore have $int (xf)^2 approx int xfp = 0$. Since $int (xf)^2$ is standard, it is equal to zero. Therefore $(xf)^2$ is identically zero, and so is $f$ (being continuous).






                        share|cite|improve this answer











                        $endgroup$


















                          1












                          $begingroup$

                          Just for fun, here's a proof in non-standard analysis (Nelson-style IST). I write $a approx b$ to mean that $a-b$ is infinitesimal.



                          It's enough to prove the result when $f$ is standard. The Weierstrass approximation theorem gives a polynomial $p$ such that $p(x) approx x f(x)$ for all $x in [0,1]$. Note $int _0 ^1 xfp = 0$, because the integral is linear.



                          Now estimate the absolute value of $int xfp - (xf)^2$. Factoring the integrand as $xf(x)cdot ( p(x) - xf(x) )$, we see that this integral is bounded above by $operatorname{sup}(xf) cdot operatorname{sup}(p-xf)$, where the sup is over all $x in [0,1]$. This is infinitesimal, since the sup of a standard continuous function on a standard compact interval is limited.



                          We therefore have $int (xf)^2 approx int xfp = 0$. Since $int (xf)^2$ is standard, it is equal to zero. Therefore $(xf)^2$ is identically zero, and so is $f$ (being continuous).






                          share|cite|improve this answer











                          $endgroup$
















                            1












                            1








                            1





                            $begingroup$

                            Just for fun, here's a proof in non-standard analysis (Nelson-style IST). I write $a approx b$ to mean that $a-b$ is infinitesimal.



                            It's enough to prove the result when $f$ is standard. The Weierstrass approximation theorem gives a polynomial $p$ such that $p(x) approx x f(x)$ for all $x in [0,1]$. Note $int _0 ^1 xfp = 0$, because the integral is linear.



                            Now estimate the absolute value of $int xfp - (xf)^2$. Factoring the integrand as $xf(x)cdot ( p(x) - xf(x) )$, we see that this integral is bounded above by $operatorname{sup}(xf) cdot operatorname{sup}(p-xf)$, where the sup is over all $x in [0,1]$. This is infinitesimal, since the sup of a standard continuous function on a standard compact interval is limited.



                            We therefore have $int (xf)^2 approx int xfp = 0$. Since $int (xf)^2$ is standard, it is equal to zero. Therefore $(xf)^2$ is identically zero, and so is $f$ (being continuous).






                            share|cite|improve this answer











                            $endgroup$



                            Just for fun, here's a proof in non-standard analysis (Nelson-style IST). I write $a approx b$ to mean that $a-b$ is infinitesimal.



                            It's enough to prove the result when $f$ is standard. The Weierstrass approximation theorem gives a polynomial $p$ such that $p(x) approx x f(x)$ for all $x in [0,1]$. Note $int _0 ^1 xfp = 0$, because the integral is linear.



                            Now estimate the absolute value of $int xfp - (xf)^2$. Factoring the integrand as $xf(x)cdot ( p(x) - xf(x) )$, we see that this integral is bounded above by $operatorname{sup}(xf) cdot operatorname{sup}(p-xf)$, where the sup is over all $x in [0,1]$. This is infinitesimal, since the sup of a standard continuous function on a standard compact interval is limited.



                            We therefore have $int (xf)^2 approx int xfp = 0$. Since $int (xf)^2$ is standard, it is equal to zero. Therefore $(xf)^2$ is identically zero, and so is $f$ (being continuous).







                            share|cite|improve this answer














                            share|cite|improve this answer



                            share|cite|improve this answer








                            edited Jan 9 '11 at 0:49

























                            answered Jan 9 '11 at 0:26









                            Matthew TowersMatthew Towers

                            7,51722446




                            7,51722446






























                                draft saved

                                draft discarded




















































                                Thanks for contributing an answer to Mathematics Stack Exchange!


                                • Please be sure to answer the question. Provide details and share your research!

                                But avoid



                                • Asking for help, clarification, or responding to other answers.

                                • Making statements based on opinion; back them up with references or personal experience.


                                Use MathJax to format equations. MathJax reference.


                                To learn more, see our tips on writing great answers.




                                draft saved


                                draft discarded














                                StackExchange.ready(
                                function () {
                                StackExchange.openid.initPostLogin('.new-post-login', 'https%3a%2f%2fmath.stackexchange.com%2fquestions%2f16831%2fnonzero-f-in-c0-1-for-which-int-01-fxxn-dx-0-for-all-n%23new-answer', 'question_page');
                                }
                                );

                                Post as a guest















                                Required, but never shown





















































                                Required, but never shown














                                Required, but never shown












                                Required, but never shown







                                Required, but never shown

































                                Required, but never shown














                                Required, but never shown












                                Required, but never shown







                                Required, but never shown







                                Popular posts from this blog

                                Human spaceflight

                                Can not write log (Is /dev/pts mounted?) - openpty in Ubuntu-on-Windows?

                                File:DeusFollowingSea.jpg